3
$\begingroup$

The fundamental group of a knot $K$ (otherwise known as the knot group) is the fundamental group of the knot complement $S^{3} \backslash K $ in $S^{3} $.

In "Virtual Knots: The State of the Art" (http://books.google.com.au/books?id=WaCJ_-MpdBYC) on page 7, it says that the fundamental group of a knot recognizes prime knots. Does this mean that the knot group of a prime knot cannot be isomorphic to the amalgamated free product of two non-trivial knot groups- and in general can the knot group of a prime knot be an amalgamated free product?

$\endgroup$
1
  • $\begingroup$ To be somewhat pedantic, the fundamental group $\pi := \pi_1(S^3 \setminus K)$ itself doesn't determine the knot complement, since mirror knot complements have isomorphic fundamental groups. However, the data $(\pi, m, l)$ does determine the knot (where $m$ and $l$ are the meridian and longitude of the knot). See mathoverflow.net/questions/35680 $\endgroup$ Jul 10, 2013 at 15:50

1 Answer 1

7
$\begingroup$

The statement on page 7 is referring to Corollary 2.1 of Gordon and Luecke's "Knots are determined by their complements."

Also a prime knot can not be an amalgamated free product of non-trivial knot groups by Corollay 2.5 of W. Thurston's "Three dimensional manifolds, Kleinian groups and hyperbolic geometry."

However, a knot group can be an amalgamated free product provided the subgroup you are amalgamating along is more complicated. For example, a knot complement with an essential, embedded quasi-Fuchsian surface has such a knot group. The basic idea of how to construct such knots goes as follows, take a genus $g$ handlebody and embed a knot in it such that the knot punctures any compressing disk in at least two points, then embedded the handlebody in $S^3$ such that the boundary of the handlebody is incompressible. This process is done in full rigor by Adams and Reid in "Quasi-Fuchsian surfaces in hyperbolic knot complements."

$\endgroup$
2
  • $\begingroup$ The link for Adams and Reid seems not to work. $\endgroup$ Jul 10, 2013 at 11:21
  • $\begingroup$ @DietrichBurde: Thank you. I fixed the broken link. $\endgroup$ Jul 10, 2013 at 13:31

Your Answer

By clicking “Post Your Answer”, you agree to our terms of service and acknowledge you have read our privacy policy.

Not the answer you're looking for? Browse other questions tagged or ask your own question.